LSAT and Law School Admissions Forum

Get expert LSAT preparation and law school admissions advice from PowerScore Test Preparation.

 Administrator
PowerScore Staff
  • PowerScore Staff
  • Posts: 8917
  • Joined: Feb 02, 2011
|
#47096
Complete Question Explanation
(The complete setup for this game can be found here: lsat/viewtopic.php?t=11700)

The correct answer choice is (D)

The question stem stipulates that G, S, and T cannot be scheduled for the fourth day. This is a rather striking group of three services because it exactly matches the three services in the third rule that cannot be scheduled for the second and third days. When a similarity like this occurs, it is extremely unlikely to be a coincidence, and you should investigate the connection further.

With G, S, and T prohibited from being scheduled on the second, third, and fourth days, G, S and T must be scheduled for the first, fifth, and sixth days. Because there are three variables and three days, that fills the appointments for those three days. The other three services—L, P, and W—cannot be scheduled for any of those three days, and must instead be scheduled for the second, third, and fourth days:
D10_game #1_#2_diagram 1.png
While G, S, and T are forced into the first, fifth, and sixth, their placement is not random. From the second rule, we know that G and S cannot be scheduled first. Thus, T must be scheduled first, and G and S must be scheduled for the fifth and sixth days, in some order:
D10_game #1_#2_diagram 2.png
Answer choice (A): While this answer choice could be true, it does not have to be true since G can be scheduled for the sixth day. Thus, this answer choice is incorrect.

Answer choice (B): While P could be scheduled for the third day, P could also be scheduled for the second or fourth days, and thus this answer choice does not have to be true, and is incorrect.

Answer choice (C): This answer is the other half of answer choice (A). While this answer choice could be true, it does not have to be true since S can be scheduled for the fifth day. Thus, this answer choice is incorrect.

Answer choice (D): This is the correct answer choice. As revealed in the discussion of the conditions in the question stem, T must be scheduled for the first day.

Answer choice (E): W could be scheduled for the second day, but it could also be scheduled for the third day, and so this answer choice does not have to be true, and is incorrect.
You do not have the required permissions to view the files attached to this post.

Get the most out of your LSAT Prep Plus subscription.

Analyze and track your performance with our Testing and Analytics Package.